Sei sulla pagina 1di 12

Argomento 10

Esercizi: suggerimenti
Ex. 1.

1. La funzione integranda è continua su [2, +∞). (Potrebbero sorgere difficoltà solo in x = 1, che
è esterno a tale intervallo). Occorre quindi esaminare la convergenza dell’integrale improprio
solo in un intorno di +∞.
Il problema è essenzialmente quello trattato nella Proposizione 10.15 della teoria: il grafico
1 1
della funzione (x−1) a si ottiene da quello di xa con una traslazione verso destra di una unità.

Quindi integrare la seconda funzione sull’intervallo [1, +∞) è come integrare la prima su
[2, +∞). Formalmente questo si verifica mediante la sostituzione t = x − 1.
1
Oppure si può osservare che (x−1) a ∼ x1a per x → +∞, quindi le due funzioni, entrambe
positive, hanno integrale improprio su un intorno di +∞ convergente per gli stessi valori di a
(Corollario 10.12), cioè per a > 1. Questa osservazione non permette però di concludere che i
due integrali impropri sono uguali, come si poteva invece ricavare dal ragionamento precedente.

2. La funzione integranda è continua su (1, 2], e può essere illimitata in un intorno destro del
punto x = 1 (per a > 0). Quindi la convergenza dell’integrale va studiata solo in tale intorno.
Anche in questo caso si può fare la seconda
R 2 delle tre osservazioni
R1 1 fatte nell’esercizio precedente:
1
con la sostituzione t = x − 1 si ottiene: 1 (x−1)a dx = 0 xa dx, che converge per a < 1.
1
La terza osservazione non ha più senso, perché NON è vero che ∼ x1a per x → 1+ .
(x−1)a
R2 1
Si può in ogni caso ignorare le osservazioni precedenti e considerare lim+ t (x−1) a dx . . .
t→1

3. La funzione integranda è continua su (1, +∞), e può essere illimitata in un intorno destro del
punto x = 1 (per a > 0). Quindi la convergenza dell’integrale improprio va discussa su un tale
intorno e su un intorno di +∞. Formalmente:
R +∞ R2 R +∞
1
f (x) dx = 1 f (x) dx + 2 f (x) dx se questi ultimi due integrali convergono. Ma per
quanto visto nei dueR esercizi precedenti il primo converge per a < 1, mentre il secondo converge
+∞
per a > 1. Quindi 1 f (x) dx non converge per nessun valore di a.

4. La funzione integranda è continua su [e, +∞); inoltre è possibile calcolare la primitiva della
funzione integranda, quindi basta applicare la definizione di integrale improprio su un intervallo
non limitato:
Z +∞ ½
1 1 £ ¤
1−a t +∞ per a<1
per a 6= 1 : dx = lim (log x) = 1
e x(log x)a t→+∞ 1−a e
a−1
per a >1

Z +∞
1
per a = 1 : dx = lim [log(log x)]te = +∞
e x log x t→+∞

5. La funzione integranda va all’infinito in x = 1 che è interno all’intervallo di integrazione, sul


resto del quale la funzione è continua. Osserviamo che x| log x| ∼ |x − 1| per x → 1,
quindi l’integrale sarà convergente per a < 1 in base al risultato dell’esercizio 1.2). Queste
considerazioni basterebbero per rispondere al quesito proposto nell’esercizio.

1
Volendo calcolare comunque il valore dell’integrale, quando esso è convergente, occorre spezzare
l’intervallo di integrazione in due parti, su ciascuna delle quali si applica poi la definizione di
integrale definito.

Z e
1 1 £ ¤
1−a t 1 £ ¤
1−a e
per a 6= 1 : dx = lim −(− log x) 1 + lim (log x) t
=
1
e
x| log x|a t→1− 1 − a e t→1+ 1 − a

½ 2
1−a
per a<1
=
+∞ per a>1
Z e
1
per a = 1 : dx = lim− [− log(− log x)]t1 + lim+ [log(log x)]et = +∞
1
e
x| log x| t→1 e t→1

(Ricordare che per 0 < t < 1 si ha log t < 0 e quindi | log t| = − log t).
Z +∞ Z e Z +∞
6. f (x) dx = f (x) dx + f (x) dx . . . (Come nel caso 3).
1 1
e e
e

Ex. 2.

1. L’integranda va all’infinito in x = 0, interno all’intervallo di integrazione. La funzione è


dispari, e l’intervallo simmetrico rispetto all’origine, quindi basta discutere l’integrale su (0, 1].
Ma sappiamo già che diverge a +∞. Quindi quello su [−1, 0) diverge a −∞ e quello su [−1, 1]
non ha senso.
2. È il caso 1.4) per a = 1
3. L’integranda va all’infinito per x → 0+ . Ma (sin1x)2 ∼ x12 per x → 0 e poiché quest’ultima
funzione ha integrale improprio divergente su (0, 1], anche l’integrale proposto diverge.
q
4. log x1 → +∞ per x → 0+ Il logaritmo non è asintotico a nessuna potenza della x, bensı̀
infinito di ordine inferiore di tutte le potenze di x (di esponente negativo, se devono andare
aq∞ per xq→ 0). Avremo quindi che, in un intorno destro abbastanza piccolo di x = 0:
log x1 < 1
,
quindi l’integrale proposto è convergente per il teorema del confronto.
x
q
Anche la relazione log x1 < x12 sarebbe corretta (in un opportuno intorno di x = 0), ma
1
sarebbe inutile ai fini dell’applicazione del teorema del confronto perché x2
ha integrale im-
proprio divergente in un tale intorno.
La scelta di √1x per maggiorare la funzione integranda è casuale ma non troppo: sarebbe andata
bene qualsiasi x1a con a < 1, perché una tale funzione ha l’integrale improprio convergente in un
intorno destro di x = 0; la scelta di a = 12 rende più immediata la verifica della disuguaglianza
usata: q
r
log x1 log t
lim+ q = lim = 0
x→0 1 t→+∞ t
x
q q
+ 1 1
quindi si ha, definitivamente per x → 0 : log x
< x
.

5. Essendo √e−x infinitesimo di ordine superiore a ogni potenza di x per x → +∞, possiamo
scrivere xe−x < x12 per x abbastanza grande, quindi l’integrale proposto converge.

2
6. Come nel caso precedente si dimostra la convergenza dell’integrale in un intorno di +∞, ma
in questo caso occorre esaminare la convergenza dell’integrale anche in un intorno destro di
x = 0, dove la funzione integranda è illimitata.
−x
Per x → 0+ si ha e√x ∼ √1x che ha integrale improprio convergente in un intorno destro di
x = 0, quindi anche l’integrale proposto è convergente.

Ex. 3.
R +∞
1. 0 sin x dx = limt→+∞ [− cos x]t0 , limite che non esiste, né finito né infinito.

2. limx→0+ sinx x = 1, quindi al funzione integranda è limitata in un intorno destro di x = 0 e


l’integrale su tale intorno è solo apparentemente improprio.
Per quel che riguarda la convergenza dell’integrale in un intorno di +∞ il discorso è complicato,
perché la primitiva non è esprimibile in termini finiti, e la funzione integranda cambia segno
infinite volte, quindi non possiamo applicare né la definizione né un teorema di confronto.
Si può comunque dimostrare che l’integrale converge.
sin x
3. x2
∼ x1 per x → 0+ , e possiamo concludere che l’integrale improprio diverge su un intorno
destro di x = 0 e quindi anche su (0, +∞), senza bisogno di discutere il problema della
convergenza dell’integrale su un intorno di +∞.
¯ x¯
Se siamo comunque interessati a tale problema, possiamo notare che ¯ sin x2
¯ ≤ 12 e quindi
x
possiamo
¯ sin x ¯ concludere che, in base al teorema di confronto, l’integrale su un intorno di +∞ di
¯ 2 ¯ è convergente e di conseguenza1 è convergente anche quello di sin2x .
x x
N.B.
¯ sin x ¯ Non1 avremmo potuto fare questo ragionamento nell’esercizio 3.2 perché, pur essendo
¯ ¯ ≤ , quest’ultima funzione ha integrale improprio divergente in un intorno di +∞.
x x

0.8
sin x 1
0.6
. tratteggiato: ±
0.4 x x
0.2

0 2 4 6 8 10 12 14 16 18 20
x
–0.2

–0.4

0.3

0.2 sin x 1
. tratteggiato: ±
y
x2 x2
0.1

0 2 4 6 8 10 12 14 16 18 20
x

–0.1

Si osservi la maggior rapidità di convergenza a zero nel secondo caso.


1
R +∞ R +∞
Vale la seguente implicazione: se a
|f (x)| dx converge, allora converge anche a
f (x) dx.

3
Ex. 4.
R 12 1 h p i12
1. 3 √ dx = lim+ 2 (x − 3) =6
x−3 t→3 t

R 1/2 1
2. 0
dx = lim+ [ log ( − log x)]1/2
t = −∞
x log x t→0

R +∞ 1 h it
1 1
3. dx = lim − 2(x−1)2 2
=
2
(x − 1)3 t→+∞ 2

R +∞ 2x + 3 h it
1 1 1
4. 1 2 dx = lim − 3x + 3(x+3)
= 4
2
(x + 3x) t→+∞ 1

R +∞ (log x)2 £1 ¤
3 t
5. dx = lim (log x) = +∞
2
x t→+∞ 3 2

R +∞ log x £ log x
¤
1 t 1 1
6. 2
dx = lim − x
− x 2
= 2
log 2 + 2
x2 t→+∞

R +∞ 1 £ √ ¤t
7. 1
√ dx = lim 2 x + 4 1 = +∞
x+4 t→+∞

R1
8. 0
log x dx = lim+ [x log x − x]1t = −1
t→0

R0 0
9. −∞
(x2 + 2x)ex dx = lim [ex x2 ]t = 0
t→−∞

R0 √ h ¡√ ¢ i0
10. ex
1 − ex dx = lim − 2
1 − e x 3 = 2
−∞ 3 t→−∞ 3
t

R1 h √ 2 it h √ i1
2
11. √1 dx = lim− 3
( 3 x) + lim+ 32 ( 3 x) = − 23 + 3
=0
−1 3 x t→0 2 t→0 2
−1 t

Rπ 1+(tan x)2
£ ¤π
12. 0
4
(tan x)2
dx = lim+ − tan1 x t4 = +∞
t→0

Ex. 5.
Z +∞ µ ¶ · µ ¶¸1 · µ ¶¸t
1 1 1 1
1. exp − dx = lim+ exp − + lim exp − = e−1 − 0 + 1 − e−1 = 1
0 x2 x t→0 x t t→+∞ x 1

Z +∞ µ ¶ · ¸1 · ¸t
cos x sin x sin x sin x
2. − 2 dx = lim+ + lim = sin 1 − 1 + 0 − sin 1 = −1
0 x x t→0 x t
t→+∞ x 1

4
Z +∞ · ¸t
−x 1 −x 1 1
3. e sin x dx = lim − e (cos x + sin x) = 0 + =
0 t→+∞ 2 0 2 2

Z µ √ ¶ · ¸t
+∞
x2 − 1 √ 1 π π
4. 1− dx = lim x− x2 − 1 − arctan √ =0−1+ = −1
1 x t→+∞ x2 − 1 1 2 2

Z π µ ¶ · µ ¶¸ π2
2 1 1 1 cos x π π
5. − dx = lim+ log x − log − = log − log 2 = log
0 x sin x t→0 sin x sin x t 2 4

Z +3 h √ i0 h √ it
x
6. √ dx = lim − 9 − x2 + lim − 9 − x2 = −3 + 0 + 0 + 3 = 0
−3 9 − x2 t→−3 t t→+3 0

Ex. 6.

1. Il parametro α è nella funzione integranda.


L’integranda è continua sull’intervallo di integrazione [1, +∞) (che non dipende dal parametro).
Infatti su tale intervallo il denominatore x(1 + log x)α non si annulla mai perché log x ≥ 0.
Quindi dobbiamo esaminare la convergenza dell’integrale improprio su un intorno di +∞ e
possiamo applicare la definizione, perché la primitiva è facilmente calcolabile:

Z +∞ · ¸t
1 1 1 1
α 6= 1 : α
dx = lim α−1
= per α > 1
1 x(1 + log x) t→+∞ 1 − α (1 + log x)
1 α−1

Z +∞
1
α=1: dx = lim [log(1 + log x)]t1 = +∞
1 x(1 + log x) t→+∞

2. Il parametro α è l’estremo sinistro dell’intervallo di integrazione.


Se α > 0 la funzione integranda è continua su [α, +∞) e occorre studiare la convergenza
dell’integrale solo in un intorno di +∞. L’esponenziale al denominatore assicura la convergenza
dell’integrale improprio (vedi esercizi 2.5 e 2.6).
Se α ≤ 0 l’intervallo di integrazione [α, +∞) contiene (a un estremo, o all’interno) il punto
1 1
x = 0 dove l’integranda va all’infinito. Ma √ x
∼ √ per x → 0; quest’ultima
3
x(1 + e ) 23x
funzione ha integrale improprio convergente in un intorno di x = 0 e quindi anche l’integrale
proposto converge.
Se α = −∞ l’integrale proposto non converge perché l’intervallo di integrazione è illimitato
1 1
a sinistra e, per x → −∞: √ x
∼ √ , funzione, quest’ultima, che ha integrale
3
x(1 + e ) 3
x
improprio divergente su un intorno di −∞.

5
Ex. 7.
L’integrale improprio che definisce la funzione Γ è convergente per ogni x > 0. Infatti:
Per t → +∞: tx−1 e−t → 0 più rapidamente di ogni potenza, quindi l’integrale improprio su un
intorno di +∞ converge per qualunque x.
1
Per t → 0+ : tx−1 e−t ∼ tx−1 = 1−x ; quest’ultima funzione ha integrale improprio su un intorno
t
di t = 0 convergente per 1 − x < 1, cioè proprio per x > 0.
Inoltre: Z +∞ Z +∞
x −t
£ ¤+∞
Γ(x + 1) = t e dt = −tx e−t 0 + x · tx−1 e−t dt = x · Γ(x)
0 0
(abbiamo usato la formula di integrazione per parti nella versione per gli integrali definiti); infatti:
£ x −t ¤+∞
−t e 0 = lim (−tx e−t ) − lim+ (−tx e−t ) = 0 − 0 = 0 ∀x > 0
t→+∞ t→0

Infine: Z +∞ £ ¤+∞
Γ(1) = e−t dt = −e−t 0 = 1
0
Applicando poi la relazione Γ(x + 1) = x · Γ(x) si ha:

per x = 1 Γ(2) = 1 · Γ(1) = 1 = 1!


per x = 2 Γ(3) = 2 · Γ(2) = 2 · 1 = 2!
per x = 3 Γ(4) = 3 · Γ(3) = 3 · 2 · 1 = 3!
per x = 4 Γ(5) = 4 · Γ(4) = 4 · 3 · 2 · 1 = 4!
...
Z x
2 2
Ex. 8. 1) F (x) = √ e−t dt
π 0

C.E.: la funzione integranda è continua su tutto R, quindi l’integrale esiste finito ∀x


Z 0
2 2
F (0) = √ e−t dt = 0
π 0
F (x) è dispari. Infatti, con la sostituzione u = −t:
Z −x Z x
2 −t2 2 2
F (−x) = √ e dt = − √ e−u du = −F (x)
π 0 π 0

lim F (x) esiste finito, quindi F ha un asintoto orizzontale per x → +∞. Infatti questa affemazione
x→+∞
R +∞ 2
equivale a dire che l’integrale improprio 0 e−t dt è convergente, e ciò è garantito dal fatto che
l’esponenziale è infinitesimo di ordine maggiore a ogni potenza (negativa) della x per x → ∞.
(Si può dimostrare, con altre tecniche, che l’asintoto è y = 1).
2 2
F 0 (x) = √ e−x , per il teorema fondamentale (Argomento 9). Quindi F 0 (x) > 0 per ogni x.
π
2 2
F 00 (x) = √ (−2x)e−x quindi F è concava verso il basso per x > 0.
π
Per il grafico vedere nelle Soluzioni.

6
Z x Z 0 Z x
2 −t2 2 −t2 2 2
2) G(x) = √ e dt = √ e dt + √ e−t dt = 1 + F (x)
π −∞ π −∞ π 0

In base a questa relazione, il grafico di G si ottiene da quello di F con una traslazione verso l’alto
di una unità. (vedere nelle Soluzioni ).
Abbiamo usato ancora la relazione:
Z 0 Z +∞
2 −t2 2 2
√ e dt = √ e−t dt = 1
π −∞ π 0
2
che non possiamo stabilire con le tecniche esposte fin’ora, in quanto la primitiva di e−t non è
esprimibile in termini finiti. Tale risultato richiede un cambiamento di variabili in un integrale
doppio e lo citiamo solo come complemento.
Ex. 9.
Z x t
e
1. F1 (x) = dt
1 t

• C.E. (0, +∞).


t Rx
Infatti la funzione integranda f (t) = et è continua su (0, +∞), quindi 1 f (t) dt è un integrale
ben definito (non improprio) per ogni x ∈ (0, +∞).
R0
Invece 1 f (t) dt non è convergente, perché f (t) ∼ 1t per t → 0+ ; quindi F1 (0) non esiste.
A maggior ragione non è definita F1 (x) per x < 0.
Rx R0 Rx
Infatti, se x < 0, 1 f (t) dt =
Rx 1 f (t) dt + 0
f (t) dt, ma già il primo di questi ultimi due
integrali non converge, quindi 1 f (t) dt non esiste (Definizione 10.7 della teoria).

• lim+ F1 (x) = −∞
x→0
in quanto, come abbiamo già detto, l’integrale improprio è divergente in un intorno di t = 0.
Il segno è dovuto al fatto che x < 1 se x → 0 (vedi convenzione pag. 1 Argomento 9).
• lim F1 (x) = +∞
x→+∞

in quanto l’integrale improprio è divergente in un intorno di +∞ perché la funzione integranda


va a sua volta all’infinito per t → +∞.

• F1 (1) = 0

ex
• F10 (x) = > 0 per ogni x > 0:
x
la F1 è sempre crescente.

x−1
• F100 (x) = ex > 0 per x > 1:
x2
c’è un flesso in x = 1.

• Per il grafico vedere nelle Soluzioni.

7
Z x −t
e
2. F2 (x) = dt
1 t

Considerazioni analoghe alle precedenti, salvo che:


• lim F2 (x) esiste finito, quindi c’è un asintoto orizzontale:
x→+∞
infatti l’integrale improprio è convergente in un intorno di +∞. Non possiamo calcolare il
limite perché la primitiva non è esprimibile in termini finiti.

x + 1 −x
• F200 (x) = − e < 0 per ogni x > 0.
x2
La funzione è concava verso il basso su tutto il C.E.

• Per il grafico vedere nelle Soluzioni.


Z x −t
e
3. F3 (x) = √3
dt
1 t
La situazione è radicalmente diversa, rispetto ai due casi precedenti, in un intorno di x = 0,
−t
dove l’integrale improprio di g(t) = e√ 3
t
converge. Ciò ha delle conseguenze sul C.E. della
funzione integrale:

• C.E. (−∞, +∞).


Rx
Infatti la funzione integranda g(t) è continua su (0, +∞), quindi 1 g(t) dt è un integrale ben
definito (non improprio) per ogni x ∈ (0, +∞).
R0
Inoltre F3 (0) = 1 g(t) dt esiste finito, ed esiste anche, per x < 0 :
Rx R0 Rx
1
g(t) dt = 1
g(t) dt + 0
g(t) dt

• lim F3 (x) = +∞
x→−∞
in quanto l’integrale improprio è divergente in un intorno di −∞ perché la funzione integranda
va a sua volta all’infinito per t → −∞.
• lim F3 (x) esiste finito, quindi c’è un asintoto orizzontale:
x→+∞
infatti l’integrale improprio è convergente in un intorno di +∞. Non possiamo calcolare il
limite perché la primitiva non è esprimibile in termini finiti.

e−x
• F30 (x)= √ > 0 per ogni x > 0, mentre F30 (x) < 0 per ogni x < 0
3
x
La derivata non è definita in x = 0, dove la funzione integranda non è continua (e quindi non
si può applicare il teorema fondamentale del calcolo integrale). Il punto x = 0 è comunque
R0
punto di minimo. Non possiamo però calcolare il valore esatto del minimo F3 (0) = 1 g(t) dt.

• lim+ F30 (x) = +∞ mentre lim F30 (x) = −∞.


x→0 x→0−
Questo determina una cuspide con tangente verticale in x = 0.

• Per il grafico vedere nelle Soluzioni.

N.B: Negli esercizi 8. e 9. abbiamo tacitamente fatto uso di un teorema che assicura la
continuità di una funzione integrale in ogni punto del suo C.E.

8
n
X n
X
1 1
Ex. 10. sn = = 1+ , quindi:
k=1
k k=2
k
Z n+1 Z n
1 1
1+ dx < sn < 1 + dx
2 x 1 x
cioè
n+1
1 + log
< sn < 1 + log n
2
ne segue, per il teorema di confronto fra limiti, che
sn
lim sn = +∞ lim = 0
n→+∞ n→+∞ n

Ex.11.
+∞
X Z +∞ h i
2 −7 2 −7 2 +∞ 2
Rn = k 5 < x 5 dx = −x− 5 = n− 5 < 10−6 per n > 1015
k=n+1
5 n 5 n

Ex.12.
+∞
X Z +∞ · ¸+∞
1 1 1 1
Rn = 3
< 3
dx = − 2
= 2
< 10−6 per n > 500
k=n+1
2k n 2x 4x n 4n

Utilizziamo la (1) (di pagina 3 degli Esercizi) per m = 1 per minorare la serie.
Per maggiorarla usiamo invece la (2) per m = 2 dopo aver osservato che
+∞
X +∞
1 1 X 1
= +
k=1
2k 3 2 k=2 2k 3

Otteniamo:
Z +∞ +∞
X Z +∞
1 1 1 1 1 1 1
= 3
dx < 3
< + 3
dx = +
4 1 2x k=1
2k 2 1 2x 2 4

L’intervallo cercato è quindi [ 14 , 34 ]


Ex.13.
+∞
X Z +∞ · ¸+∞
1 1 1 1 3)
Rn = < dx = − = < 10−3 per n > e(10
k=n+1
k(log k)2 n x(log x)2 log x n log n

Inoltre, per la (3) (di pagina 3 degli Esercizi):


Z +∞ X +∞ Z +∞
1 1 1
2
dx < s = 2
< dx
3 x(log x) k=3
k(log k) 2 x(log x)2
cioè
1 1
< s <
log 3 log 2

9
Argomento 10
Soluzioni Esercizi
Ex. 1.

1) a > 1 2) a < 1 3) nessun a

4) a > 1 5) a < 1 6) nessun a

Ex. 2.
1) non esiste 2) diverge 3) diverge

4) converge 5) converge 6) converge

Ex. 3.
1) non esiste 2) converge 3) diverge

Ex. 4.
1
1) 6 2) −∞ 3)
2
1 1 1
4) 5) +∞ 6) log 2 +
4 2 2

7) +∞ 8) −1 9) 0

2
10) 11) 0 12) +∞
3

Ex. 5.
1
1) 1 2) −1 3)
2
π π
4) −1 5) log 6) 0
2 4
Ex. 6.

1) (α − 1)−1 per α>1

2) converge per ogni α reale, ma non per α = −∞.

Ex. 7. Vedere Suggerimenti.

10
Ex. 8.

1
Z x
2 2
F (x) = √ e−t dt 0.5

π 0
–3 –2 –1 1 x 2 3

–0.5

–1
1.

2
Z x
2 2
G(x) = √ e−t dt 1.5

π −∞
1

0.5

–3 –2 –1 0 1 2 3
2. x

Ex. 9. Vedere alla pagina successiva.

Ex. 10.
n+1
1 + log < sn < 1 + log n
2
sn
lim sn = +∞ lim = 0
n→+∞ n→+∞ n

Ex.11.
n > 1015

Ex.12. · ¸
1 3
n > 500 ,
4 4

Ex.13.
3)
n > e(10
1 1
< s <
log 3 log 2

N.B. Quando di cerca il valore di un integrale o la somma (parziale o totale) di una serie il risultato
è unico, ma quando se ne cercano maggiorazioni o minorazioni (cioè approssimazioni per eccesso o
per difetto) il risultato non è univocamente determinato, ma può variare a seconda del procedimento
usato, pur rimanendo corretto.

11
Ex. 9.

Z x t 6
e
F1 (x) = dt
1 t y4

0 1x 2 3
–2
Asintoto vert.
–4
1.

x
Asintoto orizz..
0.5 1 1.5 2 2.5 3
Z x −t
0
e
F2 (x) = dt –0.5
1 t
–1

y–1.5

–2

–2.5 Asintoto vert.


–3
2.

Z x −t 1.5
e
F3 (x) = √3
dt y1
1 t
0.5 Asintoto orizz.
–1 0 1 2 3
x
–0.5

–1 Cuspide a tangente vert.

3.

12

Potrebbero piacerti anche